BÀI TẬP SỐ HỌC 15 BÀI

You might also like

Download as pdf or txt
Download as pdf or txt
You are on page 1of 6

15 BÀI TOÁN SỐ HỌC CHO HỌC SINH CHUYÊN TOÁN PTTH

1
Bài 1.
Cho n là số lẻ, n > 1. Chứng minh rằng: 3n + 1 không chia hết cho n.

http://www.math.vn-http://www.math.vn-http://www.math.vn-http://www.math.vn-http://www.math.vn-http://www.math.vn-http://www.math.vn-http://www.math.vn-http://www.math.vn

01
Lời giải
. .
Giả sử ord3 n = s ta có 32n − 1 .. n ⇒ 2n .. s
.
Nhưng vì 3n − 1 ko chia hết cho n (nếu ko thế thì 2 = 3n + 1 − (3n − 1) .. n ) nên n ko chia hết cho s vậy s = 2t
.
và ta có ngay n .. t đồng thời vì thế mà t lẻ.
. . .
Lại vì 3s − 1 .. n ⇒ 32 t − 1 = (3 t − 1)(3 t + 1) .. s = 2t bởi s = ord3 n nên s ko thể chia hết 3 t − 1 tức 3 t + 1 .. t.
Từ đây nhận thấy vai trò của t chính như n ta áp đặt tính tối đại vào là xong! 

-2
Bài 2.
Cho số nguyên tố p > 3, với mỗi k ∈ {1, 2........p − 1}, chọn duy nhất 1 số xk : kxk ≡ 1 mod p. Các số n k
n
X p−1
xác định bởi kxk = 1 + pn k ; ∀k ∈ {1, 2........p − 1}. Chứng minh rằng: k.n k ≡ mod p
k=1 2
http://www.math.vn-http://www.math.vn-http://www.math.vn-http://www.math.vn-http://www.math.vn-http://www.math.vn-http://www.math.vn-http://www.math.vn-http://www.math.vn

Lời giải

Giả sử i ∈ {1, 2, · · · , p − 1}. Với α i ∈ {1, 2, · · · , p − 1} thỏa i · α i ≡ 1( mod p) ở đây 1 ≤ i ≤ p − 1.


Rõ là (p − i)(p − α i ) ≡ 1( mod p) và i · α i = p · n i + 1 với 1 ≤ i ≤ p − 1.
pX pX
−1 −1
i · αi − 1 1 pX
−1
1 pX
−1
1 pX
−1
¶µ
Vì thế mà ta có: i · ni = i· = (i 2 · α i − i) = i2 · αi − i
i =1 i =1 p p i=1 p i=1 p i=1
p−1 p−1
vn
1 pX
−1
p−1 1 X 2 ¡ ¢ p−1 X 2 ¡ ¢ p−1
= i2 · αi − = i 2 · α i + (p − i)2 · (p − α i ) − = p2 − (2i + α i )p + (i 2 + 2i · α i ) −
p i=1 2 p i=1 2 i =1 2
p−1 p−1
2
2 p−1 2
p−1
( mod p) ≡ (i 2 + 2) −
X X
≡ (i + 2i · α i ) − ( mod p) (bởi i · α i ≡ 1( mod p)
iµ=1 2 i =1 2
p−1 p+1 p−1 p+1
¶µ ¶ µ ¶µ ¶
p p
2 2 (p − 1) p − 1 2 2
≡ +2· − ( mod p) Để ý là ≡ 0( mod p); p > 3
6 2 2 6
n n(n + 1)(2n + 1) p − 1
i2 =
X
Do đó ta thấy là: ≡ ( mod p) 
th.

i =1 6 2
Bài 3.
Cho f (x) : N∗ → N∗ thỏa mãn f (x y) = f (x). f (y) với mọi x; y, thỏa mãn (x; y) = 1. f (x + y) = f (x) + f (y)
với x; y là các số nguyên tố tùy ý. Tính f (2); f (3); f (2009)
http://www.math.vn-http://www.math.vn-http://www.math.vn-http://www.math.vn-http://www.math.vn-http://www.math.vn-http://www.math.vn-http://www.math.vn-http://www.math.vn

Lời giải

Có f (6) = f (3) + f (3) = f (2). f (3) vậy f (2) = 2


Lại có f (3) = f (5) − f (2) = f (7) − f (2) − f (2) = f (12) − f (5) − 4 = f (12) − f (3) − f (2) − 4
= f (4). f (3) − f (3) − 6 = ( f (2) + f (2). f (3) − f (3) − 6 = 4. f (3) − f (3) − 6 ⇒ f (3) = 3
ma

f (2009) = f (49). f (41); f (4). f (11) = f (44) = f (3) + f (41); f (4) = f (2) + f (2) = 4;
f (11) = f (14) − f (3) = f (2). f (7) − f (3) Còn f (7) = f (2) + f (5) = f (2) + f (2) + f (3) = 7
Vậy f (11) = 11 và f (41) = 41
f (49) = f (2) + f (47) = f (2) + f (50) − f (3) = f (2) + f (2) f (25) − f (3)
f (25) = f (2) + f (23) = f (2) + f (26) − f (3) = f (2) + f (2). f (13) − f (3)
f (13) = f (11) + f (2) = 13 ⇒ f (25) = 25 ⇒ f (49) = 49
Vậy f (2009) = 2009 

1
Bài 4.
Hãy khẳng định hoặc phủ định mệnh đề sau:

1
Với mọi cặp số vô tỷ s; r luôn tồn tại m; n ∈ N sao cho [sm] = [rn]
http://www.math.vn-http://www.math.vn-http://www.math.vn-http://www.math.vn-http://www.math.vn-http://www.math.vn-http://www.math.vn-http://www.math.vn-http://www.math.vn

Lời giải

01
Ta sẽ khẳng định mệnh đề. Với mọi cặp số thực s; r luôn tồn tại m; n ∈ N sao cho [sm] = [rn].
Ta chỉ cm với s 6= 0 và r 6= 0.
Với hai số thực s, r chọn số số tự nhiên k > ·max
¸ | s |, | r |,
i i i i
· ¸
ta xét hai dãy số (s i ); (r i ) như sau: s i = − và r i = − với 0 ≤ i ≤ k,
s r s r
như vậy ta có k2 + 1 cặp số (s i , r i ) và tương ứng với nó là cặp số nguyên ([ks i ], [kr i ]) vơi mọi 0 ≤ i ≤ k.
Mặt khác 0 ≤ s i , r i < 1 nên chỉ có k2 cặp ([ks i ], [kr i ]),
do đó tồn tại hai cặp (s i 1 , r i 1 ); (s i 2 , r i 2 ) với 0 ≤ i 1 < i 2 ≤ k sao cho [ks i 1 ] = [ks i 2 ]; [kr i 1 ] = [kr i 2 ].

-2
1 1
Suy ra | ks i 1 − ks i 2 | = | ks i 1 − ks i 2 | < 1 ⇒ | s i 1 − s i 2 | < , tương tự | r i 1 − r i 2 | < .
| s| |r|
i2 i1 i2 i1
· ¸ · ¸ · ¸ · ¸
Chọn m = − ; n= − và đặt p = i 2 − i 1
s s s s
¯p ¯ 1
ta có: ¯ − m¯ = |s i 1 − s i 2 | < ⇒ | p − ms| < 1 ⇒ p − 1 < ms < p ⇒ [ms] = p;
¯ ¯
¯p
s | s|
¯ ¯ ¯ 1
¯ − n¯ = ¯ r i 1 − r i 2 ¯ < ⇒ | p − nr | < 1 ⇒ p − 1 < nr < p ⇒ [nr] = p.
¯ ¯
r |r|
Vậy tồn tại hai số tự nhiên m, n để [ms] = [nr]. 
Bài 5. Olympic BaLan 1986
Chứng minh rằng với mỗi n ∈ N, n ≥ 3 thì n! luôn biểu diễn được bằng tổng của n ước số khác nhau
của nó.
http://www.math.vn-http://www.math.vn-http://www.math.vn-http://www.math.vn-http://www.math.vn-http://www.math.vn-http://www.math.vn-http://www.math.vn-http://www.math.vn
vn
Lời giải
nX
−1 n!
Viết n! = (n − 1)! + là xong!
k=1 k(k + 1)
Bài 6.
Tìm số nguyên tố p và các số nguyên dương x, y thỏa mãn phương trình x3 + y3 = p4 .

http://www.math.vn-http://www.math.vn-http://www.math.vn-http://www.math.vn-http://www.math.vn-http://www.math.vn-http://www.math.vn-http://www.math.vn-http://www.math.vn

Lời giải
th.

Có ngay x + y; x2 − x y + y2 là ước của p4 do đó x + y; x2 − x y + y2 ∈ {1; p; p2 ; p3 ; p4 }


và hễ x + y = p k thì x2 − x y + y2 = p4−k .
x2 + y2 (x + y).(x + y)
Lại thấy (x + y)2 > x2 − x y + y2 ≥ ≥ do đó cả x; y đều phải bé hơn 4.
2 4
Thay vào thử thấy có mỗi x = y = p = 2 thỏa.
Bài 7.
Cho x; y; n; p ∈ N∗ thỏa x n + yn = p k biết p; n; k > 1 và p; n cùng lẻ, p là số nguyên tố. Chứng minh
rằng n là lũy thừa của p.
http://www.math.vn-http://www.math.vn-http://www.math.vn-http://www.math.vn-http://www.math.vn-http://www.math.vn-http://www.math.vn-http://www.math.vn-http://www.math.vn
ma

Lời giải

Không mất tính tổng quát ta chỉ xét trường hợp (x; p) = (y; p) = 1
Có x + y là ước của p k đồng thời x + y > 1 do đó x + y = p l với 0 < l ≤ k
n .
Ta thấy p k = x n + yn = x n + (p l − x)n = (p l )2 . (− x)n− t .p l ( t−2) C nt + n.x n−1 .p l .. p l +1
X
t=2
. .
⇒ n.x n−1 .p l .. p l +1 ⇒ n.. p ⇔ n = p.n 1 .

2
Lại lặp lại suy luận này với phương trình nghiệm nguyên có nghiệm là:
.
X n1 + Y n1 = p k với X = x p ; Y = y p ta lại có n .. p .... từ đây có ĐPCM!

1
1
Bài 8.
Tính trung bình cộng của tất cả các số tự nhiên n thỏa mãn n có 2010 chữ số mà các chữ số đều
thuộc tập {1, 2, 3, 4, 5, 6, 7, 8} đồng thời n chia hết cho 99999.
http://www.math.vn-http://www.math.vn-http://www.math.vn-http://www.math.vn-http://www.math.vn-http://www.math.vn-http://www.math.vn-http://www.math.vn-http://www.math.vn

01
Lời giải

Có r ∈ {1, 2, 3, 4, 5, 6, 7, 8} ⇔ 9 − r ∈ {1, 2, 3, 4, 5, 6, 7, 8} và rõ ràng 9 − r! = r .


Do đó hễ mà n = a 1 a 2 ..a 2010 là số có 2010 chữ số mà các chữ số đều thuộc tập {1, 2, 3, 4, 5, 6, 7, 8}
thì n0 = f (A) = b1 b2 ..b2010 với b i = 9 − a i cũng là số có 2010 chữ số
. .
mà các chữ số đều thuộc tập {1, 2, 3, 4, 5, 6, 7, 8} đồng thời n! = n0 và n + n0 = 99...9..99999 (để ý 2010..5).
Thêm nữa hễ n1 ! = n2 thì rõ ràng n01 = f (n1 )! = n02 = f (n2 ).

-2
Tất cả lý luận trên cho thấy tập các số tự nhiên n thỏa mãn: n có 2010 chữ số mà các chữ số đều thuộc
tập {1, 2, 3, 4, 5, 6, 7, 8} đồng thời n chia hết cho 99999 được phân hoạch thành từng cặp rời nhau mà tổng
các cặp đó là 99...9.
1 102011 − 1
Vậy trung bình cộng cần tính là 99...9 = .
2 2
Bài 9.
Chứng minh rằng ∀n ∈ N∗ thì giữa n2 và (n + 1)2 luôn tồn tại ba số tự nhiên phân biệt a; b; c thỏa
.
a2 + b2 .. c.
http://www.math.vn-http://www.math.vn-http://www.math.vn-http://www.math.vn-http://www.math.vn-http://www.math.vn-http://www.math.vn-http://www.math.vn-http://www.math.vn

Lời giải

Chọn a = n2 + 2, b = n2 + n + 1, c = n2 + 1 thì hiển nhiên n2 < a, b, c < n2 + 2n + 1.


Hơn nữa
vn
a2 + b2 = (n2 + 1 + 1)2 + (n2 + 1 + n)2 = (n2 + 1)2 + 2(n2 + 1) + (n2 + 1)2 + 2n(n2 + 1) + n2 + 1 = (n2 + 1).m.
Bài 10.
Cho các số nguyên dương a ≥ b ≥ c và d thỏa mãn các điều kiện sau:
* abc = d 3 .
* Số a + b + c − d là một ước số nguyên tố của số ab + bc + ca − d 2 Chứng minh rằng b = d .
http://www.math.vn-http://www.math.vn-http://www.math.vn-http://www.math.vn-http://www.math.vn-http://www.math.vn-http://www.math.vn-http://www.math.vn-http://www.math.vn

Lời giải
th.

Từ giả thiết có ngay c ≤ d ≤ a và p = a + b + c − d | d.(ab + bc + ca − d 2 ) − d 2 .(a + b + c − d)


= d 3 − (a + b + c)d 2 + (ab + bc + ca)d − d 3 = d 3 − (a + b + c)d 2 + (ab + bc + ca)d − abc = (d − a).(d − b).(d − c).
Do p = a + b + c − d là số nguyên tố nên buộc p phải là ước của 1 trong các số d − a; d − b; d − c.
- Nếu a = c thì a = b = c = d kết luận của bài toán là hết sức tầm thường.
| d − a| a−d
- Nếu a > c p ko thể là ước của d − a vì 0 < = < 1.
p a+b+c−d
p lại càng không thể là ước của d − c đơn giản vì theo bdt Cauchy a + b + c − d > 3d − d = 2d > d − c > 0
Vậy chỉ có thể p là ước của d − b
.
ma

lại cũng vì p > 2d > d − b nên nếu d > b thì p > d − b > 0 nên ko thể có d − b.. p.
Còn nếu b > d thì p = b − d < b − d + a + c = p cũng ko thể thỏa. Tóm lại: b = d 
Bài 11.
Cho dãy số nguyên (a n ) xác định bởi: a 0 = 1; a 1 = −1; a n = 6a n−1 + 5a n−2 với mọi n ≥ 2.
Chứng minh rằng a 2012 − 2010 chia hết cho 2011
http://www.math.vn-http://www.math.vn-http://www.math.vn-http://www.math.vn-http://www.math.vn-http://www.math.vn-http://www.math.vn-http://www.math.vn-http://www.math.vn

Lời giải

3
p p
Phương trình đặc trưng của dãy đã cho là: x2 − 6x − 5 = 0 có hai nghiệm là 3 − 14; 3 + 14 do đó dễ dàng
có được công thức số hạng tổng quát
p ¢ của dãy ¢là: ¡

1
¡ ¡ p p ¢¡ p ¢ n n
7 − 2 14 3 + 14 + 7 + 2 14 3 − 14
an = = − u n − 2vn
14
Với: ¡ p ¢n−1 ¡ p ¢n−1 ¡ p ¢n−1 ¡ p ¢n−1
3 + 14 + 3 − 14 3 + 14 − 3 − 14
un = ; vn = p

01
2 2 14
Sử dụng khai triển Newton ta có được:
1005 1005
2k
32011−2k 14k = 32011 +
2k
32011−2k 14k
X X
u 2012 = C 2011 C 2011
k=0 k=1 Ã
2 k−1
!
2k
C 2010 ..
Do 1 < 2k < 2011 với 1 ≤ k ≤ 1005 vậy nên số nguyên C2011 = 2011 . 2011 vì 2011 là số nguyên tố.
2k
Vậy u2012 ≡ 3 (mod 2011) do 32011 ≡ 3 (mod 2011) (định lý Fermat bé).
Cũng theo khai triển Newton thì:

-2
P1005 2 k−1 2012−2 k
v2012 = 141005 + k=1
C 2011 3 14k−1
2 k−2
à !
2 k−1
C 2010 ..
Để ý rằng: C2011 = 2011 . 2011 với k ∈ {1; 2; · · · 1005} do tính nguyên tố của 2011.
2k − 1
14
¶ µ
Thế nên v2012 ≡ 141005 (mod 2011) Do = −1 nên ta có v2012 ≡ −1 (mod 2011)
2011
Vậy a 2012 − 2010 ≡ −3 − 2(−1) − 2010 ≡ 0 (mod 2011) 
Bài 12.
Cho p là số nguyên tố và dãy tăng các số nguyên dương {r i }m i =1
nhỏ hơn p thỏa: r m
i
≡ 1 (mod p) ∀ i ∈
m
{1; 2; · · · ; m}. Chứng minh rằng: x − 1 ≡ (x − r 1 )(x − r 2 ) · · · (x − r m ) (mod p) ∀ x ∈ Z.
http://www.math.vn-http://www.math.vn-http://www.math.vn-http://www.math.vn-http://www.math.vn-http://www.math.vn-http://www.math.vn-http://www.math.vn-http://www.math.vn

Lời giải

Bài này dùng khai triển Abel: f (x) = x m − 1 = b0 (x − r 1 )...(x − r m ) + ... + b m−1 (x − r 1 ) + b m
vn
So sánh hệ số x m suy ra b0 = 1. Từ giả thiết suy ra:
f (r 1 ) = r 1m − 1 ≡ 0 (mod p) ⇒ b m ≡ 0 (mod p)
f (r 2 ) = r 2m − 1 ≡ 0 (mod p) ⇒ b m−1 ≡ 0 (mod p)
.
.
.
f (r m ) = r m
m − 1 ≡ 0 (mod p) ⇒ b 1 ≡ 0 (mod p)
th.

Suy ra: x m − 1 ≡ (x − r 1 )(x − r 2 ) · · · (x − r m ) (mod p) ∀ x ∈ Z. 


Bài 13.
Cho p là số nguyên tố lẻ.
Chứng minh rằng không tồn tại các số nguyên x, y thỏa mãn hệ thức: x p + y p = p[(p − 1)!] p .
http://www.math.vn-http://www.math.vn-http://www.math.vn-http://www.math.vn-http://www.math.vn-http://www.math.vn-http://www.math.vn-http://www.math.vn-http://www.math.vn

Lời giải

Giả sử có x; y ∈ Z thỏa yêu p((p − 1)!) p = x p + y p thì theo định lý Fermat bé thì:
p((p − 1)!) p = x p + y p ≡ x + y mod p
ma

Vì thế sẽ dẫn đến biểu diễn x = k p − y với k ∈ Z. Lúc ấy do C pp−1 = p nên:


P p−2 j
x p + y p = (k p − y) p + y p = j =0
C p (k p) p− j (− y) j + k p2 y p−1 ≡ 0 mod p2
. . .
Vậy p((p − 1)!) p .. p2 tức ((p − 1)!) p .. p dẫn đến (p − 1)! .. p
Điều không thể xảy ra này dẫn ta đến sự khẳng định cho những gì cần chứng minh 
Bài 14.
.
Chứng minh tồn tại x; y; z : ax2 + b y2 + cz2 .. p

http://www.math.vn-http://www.math.vn-http://www.math.vn-http://www.math.vn-http://www.math.vn-http://www.math.vn-http://www.math.vn-http://www.math.vn-http://www.math.vn

4
Lời giải

1
Cách 1
Với p nguyên tố
½ ta có nhận xét: ¾
p−1 p−1
* Tập số i 2 : i = 1; . . . ; từng đôi không đồng dư mod p.
2 2
* i 2 = (p − i)2 mod p

01
p−1 p−1
Do đó tập T = {1; 2; . . . ; p − 1} chứa đúng
số chính phương mod p và số không là chính phương
2 2
mod p. Gọi L là tập các số chính phương mod p của T . Lúc ý dễ dàng kiểm tra:
aL = L ∀ a ∈ L và aL = T \ L ∀ a ∉ L
(Thực ra L là nhóm con của nhóm nhân Z p và có hai loại lớp kề L và T \ L )
Ta cần chỉ ra có thể chọn trong mỗi tập: aL ∪ {0}, bL ∪ {0}, cL ∪ {0} một phần tử để tổng của chúng là bội
p.
Xét ba tập aL, bL, cL có vài trường hợp sau:

-2
* Có hai tập là L còn tập còn lại là T \ L chẳng hạn aL = bL = L; cL = T \ L lúc ý:
* Nếu p − 1 ∈ L thì 1 + p − 1 + 0 thỏa mãn
* Nếu p − 1 ∈ T \ L thì 1 + 0 + p − 1 thỏa mãn.
* Có hai tập là T \ L còn tập còn lại là L chẳng hạn aL = bL = T \ L; cL = L lúc ý gọi u là phần tử của
T \ L thì:
* Nếu p − u ∈ L thì u + 0 + p − u thỏa mãn
* Nếu p − u ∈ T \ L thì u + p − u + 0 thỏa mãn.
p−1 p−3 p+1 p−1 p−1
µ ¶ µ ¶
* Ba tập đó đểu kiểu L, lúc ý xét cặp (1; p − 2), (2; p − 3), . . . , ; ), ; có:
2 2 2 2 2
* Nếu có một cặp (i; p − 1 − i) cùng thuộc L thì tổng 1 + i + (p − 1 − i) thỏa mãn
* Ngược lại, nếu các cặp (i; p − 1 − i) đều có không quá một số thuộc L thì cặp cuối cùng ít nhất có một
vn
số thuộc L:
* Nếu là p − 1 ∈ L thì tổng 1 + (p − 1) + 0 thỏa mãn
p−1 p−1 p−1
* Nếu là ∈ L thì tổng 1 + + thỏa mãn.
2 2 2
* Ba tập đó cùng kiểu T \ L, gọi u là phầnµ tử của T \ L.
(p − 1)u (p − 1)u

Xét các cặp (u; (p − 2)u), (2u; (p − 3)u), . . . , ; .
2 2
* Nếu có một cặp (iu; (p − 1 − i)u) cùng thuộc T \ L thì tổng u + iu + (p − i − 1)u thỏa mãn
* Ngược lại thì trong cặp cuối cùng có ít nhất một số thuộc T \ L:
th.

* Nếu (p − 1)u ∈ T \ L thì tổng u + (p − 1)u + 0 thỏa mãn


(p − 1)u (p − 1)u (p − 1)u
* Nếu ∈ T \ L thì tổng u + + thỏa mãn
2 2 2
Tóm lại bài toán được chứng minh 
Cách 2
Giả dụ có một trong ba số a; b; c kia mà là bội của p thì điều ta cần khẳng định là hết sức tầm thường.
Thí dụ hễ a là bội của p thì chọn x = 1 còn y = z = 0 là xong.
Khi cả a; b; c đều chả là bội của p lại xét hai khả năng sau:
1) Nếu tồn tại x; y không đồng thời là bội của p mà p | ax2 + b y2 thế thì chọn z = 0 là xong.
ma

2) Nếu không tồn tại x; y (không đồng thời là bội)của p(mà p | ax2 + b y2 khi đó xét)hai tập sau:
p−1 p−1
A = ak2 : k = 1; 2; · · · ; , B = − bl 2 − c : l = 1; 2; · · · ;
2 2
Rất dễ dàng thấy là các phần tử của A không có đồng dư với nhau khi chia cho p và tất nhiên trong A
không có bội của p. Tình hình bên tập B kia cũng vậy.
* Hễ có một phần tử của A mà đồng dư với một phần tử của B thì xong ngay.
* Hễ không thể chọn ra từ mỗi tập một phần tử để chúng có cùng số dư khi chia cho p. Thế thì dẫn đến:
P P
a∈ A a + b∈ A b ≡ 1 + 2 + · · · + p − 1 mod p

5
Khổ cái điều này dẫn đến:
p2 − 1 p−1 p−1
p−c (a − b)
≡ p ≡ 0 mod p

1
24 2 2
p−1
Điều này thì đúng thế nào nổi do c không thể là bội của p 
2
Bài 15.
x2n−1 + x2n−2
Cho dãy số (xn )n được xác định như sau: x1 = a, x2 = b, xn = , ∀ n ≥ 3. Với a, b > 1 là các số

01
xn−1 + xn−2
nguyên tố phân biệt. Chứng minh rằng xn không phải là số nguyên với mọi n ≥ 3.
http://www.math.vn-http://www.math.vn-http://www.math.vn-http://www.math.vn-http://www.math.vn-http://www.math.vn-http://www.math.vn-http://www.math.vn-http://www.math.vn

Lời giải

Dãy trên dĩ nhiên là dãy con của dãy các số hữu tỷ dương, vậy nên ta có thể đặt:
an
xn = ; a n ; b n ∈ Z+ ; GCD (a n ; b n ) = 1 ∀ n ∈ Z+
bn
Vì xn > 1 ∀ n ∈ Z+ vậy nên:

-2
a n > b n ≥ 1 ∀ n ∈ Z+ (*).
Việc của chúng ta là đi chứng minh:
b n > 1 ∀ n ≥ 3; n ∈ Z+ và GCD(a n ; a n+1 ) = 1 ∀ n ∈ Z+
Điều này sẽ được khẳng định nhờ phép quy nạp như sau:
Thật vậy ta thấy rằng với n = 1; 2; 3 thì mọi thứ là rõ ràng
Bây giờ ta giả sử điều cần minh tỏ đã đúng tới n − 1 khi đó ta thấy:
an a2n−1 + a2n−2
=
b n b n−1 b n−2 (a n−1 + a n−2 )
Từ đẳng thức đó và việc GCD (a n ; b n ) = 1 ta dẫn đến:
a n | a2n−1 + a2n−2 ; b n | b n−1 b n−2 (a n−1 + a n−2 ).
Từ GCD(a n−2 ; a n−1 ) = 1 ta có ngay GCD(a n ; a n−1 ) = 1.
a n−1 + a n−2
½ ¾
vn
Giờ gọi p n là ước nguyên tố nhỏ nhất của a n−1 + a n−2 và d n = max p n ; .
pn
Bởi vì (∗) và GCD(a n−2 ; a n−1 ) = 1 nên d n > 2.
Mặt khác
GCD a2n−1 + a2n−2 ; a n−1 + a n−2 là ước của GCD a2n−1 + a2n−2 ; a2n−1 − a2n−2 ≤ 2 < d n (vì GCD(a n−2 ; a n−1 ) = 1).
¡ ¢ ¡ ¢

Vậy nên GCD(a n ; d n ) = GCD(a2n−1 + a2n−2 ; d n ) = 1 từ đó d n | b n dẫn đến b n > 1 


th.
ma

You might also like